Position Operator in Momentum Space?

In summary, the student attempted to solve a homework problem from Townsend's QM book, but made a mistake in assuming that the position operator takes that form in momentum space. They need to use closure relation in position space instead. Once they do this, they are able to solve the problem using the derivative of the integration variable.
  • #1
Crush1986
207
10

Homework Statement


So, I'm doing this problem from Townsend's QM book
6.2[/B]
Show that [tex] <p|\hat{x}|\psi> = i\hbar
\frac{\partial}{\partial p}<p|\psi>
[/tex]

Homework Equations


[tex] |\psi(p)> = \int_\infty^{-\infty} dp |p><p|\psi> [/tex]

The Attempt at a Solution


So,
[tex] <p|\hat{x}|\psi> [/tex]
[tex] = <p|\hat{x} \int_\infty^{-\infty} dp' |p'><p'|\psi> [/tex]
[tex] = \hat{x} \int_\infty^{-\infty} dp' <p|p'> <p'|\psi> [/tex]
[tex] = \hat{x} \int_\infty^{-\infty} dp' \delta (p-p') <p'|\psi> [/tex]
[tex] = \hat{x} <p|\psi> [/tex]

and here I've assumed, correctly I've found, that [tex] \hat{x} = i\hbar \frac{\partial}{\partial p}[/tex]
However, knowing my professor if I just write that and say "tada." I'm not going to get a good grade on this problem set.

How would I prove that the position operator takes that form in momentum space? I've found a proof that uses the more traditional notation with integrals and the like but I've read that it is pretty nice when done in Dirac notation, I'd appreciate any help/hint.

Thank you.
 
Physics news on Phys.org
  • #2
I wonder how you can claim to have found the answer correctly when there is a mistake in assuming that ##\langle p|\hat x | p' \rangle = \hat x \langle p|p' \rangle##.
 
  • #3
blue_leaf77 said:
I wonder how you can claim to have found the answer correctly when there is a mistake in assuming that ##\langle p|\hat x | p' \rangle = \hat x \langle p|p' \rangle##.
Hrm, thought I had seen something similar done in my book. Welp, then I guess I'm completely lost. I suppose it was done only after the momentum operator was written in position space. I cannot pull it out like that then until it has been rewritten?
 
  • #4
May be your memory deceived you. Certainly ##\delta(p-p')## is not equal to ##\partial_p \delta(p-p')##.
It's easier if you use closure relation in position space rather than that in momentum space. Use it in ##\langle p |\hat x| \psi\rangle##.
 
  • #5
Ok, I originially was trying this but wasn't sure what to do after,

[tex]
\langle p |\hat x| \psi\rangle [/tex]

[tex] = \int_\infty^{-\infty} \langle p |\hat x| x \rangle \langle x | \psi \rangle dx [/tex]
[tex] = \int_\infty^{-\infty} x \langle p | x \rangle \langle x | \psi \rangle dx [/tex]
I think that,
[tex] \langle p|x \rangle = \frac{1}{\sqrt{2 \pi \hbar}}e^{\frac{-ipx}{\hbar}} [/tex]
and,
[tex] \langle x|\psi \rangle = \int_\infty^{-\infty} dp \frac{1}{\sqrt{2 \pi \hbar }} e^{\frac{ipx}{\hbar}} \langle p | \psi \rangle [/tex]

If I plug those in though I get a nightmare! Is this the right path?
 
  • #6
You don't need the last one actually, just the expression for ##\langle p|x\rangle##. Upon plugging in this to ##
\int_\infty^{-\infty} x \langle p | x \rangle \langle x | \psi \rangle dx
##
You will get terms like ##x \exp{(ipx/\hbar)}##. How can you get this expression from ## \exp{(ipx/\hbar)
}## alone?
 
  • #7
Hrm, I'm still stuck. I have no idea what to do with something like this!?

[tex] \frac{1}{\sqrt{2 \pi \hbar}} \int_\infty^{-\infty} x e^{\frac{-ipx}{\hbar}} \langle x | \psi \rangle dx [/tex]

I don't know if it's because I'm just starting with all these integrals with Dirac notation or what, but I'm just so confused with stuff like this right now.

integration by parts? it's the [tex] \langle x | \psi \rangle [/tex] that has me wierded out.
 
  • #8
##\frac{\partial}{\partial p} \exp(-ipx/\hbar) = ??##
 
  • #9
That is [tex] -ix/\hbar exp(-ipx/\hbar) [/tex] I'm still not sure where that is going though. I see that that is part of my integrand.
 
  • #10
Crush1986 said:
That is [tex] -ix/\hbar exp(-ipx/\hbar) [/tex] I'm still not sure where that is going though. I see that that is part of my integrand.
Yes, that means you can replace that with the expression involving the derivative in your integral.
 
  • #11
Wait, so does that just basically say... that [tex] x = i \hbar \frac{\partial}{\partial p} [/tex]
 
  • #12
Crush1986 said:
do you just divide out the exponentials
Definitely no. What I meant is, since $$
\frac{\partial}{\partial p} \exp(-ipx/\hbar) = \frac{-ix}{\hbar} \exp(-ipx/\hbar)$$
you can replace the entire expression ##x \exp(-ipx/\hbar)## with ##i\hbar \frac{\partial}{\partial p} \exp(-ipx/\hbar) ##.
 
  • #13
Ok let me see that. I thought for a second I could just solve that expression for x. But then I wasn't sure how on Earth you could just say that x was the position operator haha.
 
  • #14
I'm still stuck with an integral I just don't know what to do with.

[tex] \frac{i\hbar}{\sqrt{2\pi\hbar}} \int_\infty^{-\infty} \frac{\partial}{\partial p} e^{\frac{-ipx}{\hbar}} \langle x|\psi \rangle dx [/tex]
 
  • #15
The integration variable is ##x## while that partial derivative operates on ##p##-dependent function. What can you do with the derivative? (If you don't know what to do you might want to review your calculus lecture).
 
  • #16
wait this looks like [tex] ih \frac{\partial}{\partial p} \psi(p) [/tex]
 
  • #17
Yes, that should be in the final step. But I guess you also need to know where to move the derivative. Check "Leibniz rule".
 
  • #18
oh... and that is it? yah?
 
  • #19
I mean, it's not the variable being integrated so it gets pulled out and turns into a regular derivative, I thought.
 
  • #20
Crush1986 said:
I mean, it's not the variable being integrated so it gets pulled out and turns into a regular derivative, I thought.
Yes it can be pulled out of the integral but it's better to let it stay being a partial derivative. Later for more than one dimension you will see that the position and momentum operators cannot be reduced to ordinary derivative.
 
  • #21
blue_leaf77 said:
Yes it can be pulled out of the integral but it's better to let it stay being a partial derivative. Later for more than one dimension you will see that the position and momentum operators cannot be reduced to ordinary derivative.
Thanks for all your help! That makes sense.
 

1. What is the position operator in momentum space?

The position operator in momentum space is a mathematical representation of the position of a particle in terms of its momentum. It is used in quantum mechanics to describe the position of a particle in a system.

2. How is the position operator related to the momentum operator?

The position operator and the momentum operator are related through the Heisenberg uncertainty principle. This principle states that the more precisely the position of a particle is known, the less precisely its momentum can be known, and vice versa.

3. How is the position operator represented mathematically?

The position operator in momentum space is represented by the variable x, which is multiplied by the momentum operator p in the equation: x = iℏ∂/∂p. Here, ℏ is the reduced Planck's constant and ∂/∂p represents the partial derivative of the momentum operator.

4. What is the significance of the position operator in momentum space?

The position operator in momentum space is significant because it allows us to calculate the probability of finding a particle at a certain position in a quantum system. It is also used in the Schrödinger equation to describe the time evolution of a quantum system.

5. How does the position operator in momentum space relate to the physical position of a particle?

The position operator in momentum space does not directly relate to the physical position of a particle. It is a mathematical representation that describes the probability of finding a particle at a certain position in a quantum system. The physical position of a particle can only be determined through measurement.

Similar threads

  • Introductory Physics Homework Help
Replies
1
Views
889
Replies
3
Views
824
  • Advanced Physics Homework Help
Replies
10
Views
582
  • Quantum Physics
2
Replies
56
Views
3K
Replies
24
Views
527
  • Introductory Physics Homework Help
Replies
4
Views
2K
  • Advanced Physics Homework Help
Replies
4
Views
948
  • Introductory Physics Homework Help
Replies
28
Views
368
  • Advanced Physics Homework Help
Replies
1
Views
1K
Replies
10
Views
1K
Back
Top